Bài tập về phương pháp Lagrange

Tóm tắt nội dung tài liệu

  1. PHƯƠNG PHÁP NHÂN TỬ LAGRANGE1 -METHOD OF LAGRANGE MULTIPLIERS Trần Trung Kiên TP. Hồ Chí Minh- Ngày 30 tháng 9 năm 2012 Phương pháp nhân tử Lagrange (sẽ được học trong chương trình toán cao cấp của bậc đại học) khá hiệu quả trong những bài toán cực trị có điều kiện ràng buộc ngoài ra còn có thể dùng để tìm điều kiện xảy ra dấu bằng của bất đẳng thức.  Định nghĩa K Cực trị (cực đại hoặc cực tiểu) có điều kiện của hàm hai biến z = f (x; y) là cực trị của hàm này với điều kiện là các biến x, y phải thỏa ràng buộc bởi phương trình ϕ(x; y) = 0. Để tìm cực trị có điều kiện của hàm z = f (x; y) khi hiện hữu phương trình ràng buộc ϕ(x; y) = 0, người ta thiết lập một hàm bổ trợ là hàm Lagrange: L(x; yλ) = f (x; y)+λϕ(x; y), trong đó λ là một nhân tử hằng chưa xác định, gọi là nhân tử Lagrange. Điều kiện cần của cực trị là hệ ba phương trình. L0x (x; y; λ) = fx0 (x; y) + λϕ0x (x; y) = 0 L0y (x; y; λ) = fy0 (x; y) + λϕ0y (x; y) = 0 ϕ(x; y) = 0 Giải hệ trên ta tìm được nghiệm là x0 ; y0 ; λ0 . Vấn đề tồn tại và đặc tính của cực trị có điều kiện được giải bằng cách xét dấu vi phân cấp 2 của hàm Lagrange tại điểm P0 (x0 ; y0 ) và λ0 - nghiệm của hệ phương trình trên. P0 (x0 ; y0 ) là điểm dừng của hàm L. d2 L = L00xx dx2 + 2L00xy dxdy + L00yy dy 2 T Trong đó dx; dy thỏa mãn ràng buộc biểu thị bằng phương trình ϕ0x dx + ϕ0y dy = 0(dx2 + dy 2 6= 0) Cụ thể xét hàm f (x; y) đạt cực đại có điều kiện nếu d2 L < 0 và đạt cực tiểu có điều kiện nếu d2 L > 0 tại điểm dừng P0 (x0 ; y0 ) và nhân tử λ0 .  Các bước cơ bản của phương pháp nhân tử Lagrange 1. Phát biểu bài toán dưới dạng mô hình toán học. Cực đại hoặc cực tiểu của hàm z = f (x; y) với điều kiện ràng buộc ϕ(x; y) = 0 2. Thiết lập hàm Lagrange L(x; y; λ) = f (x; y) + λϕ(x; y) 3. Tìm điểm dừng của L, tức là giải hệ phương trình  0  Lx (x; y; λ) = 0 T  L0y (x; y; λ) = 0   0 Lλ (x; y; λ) = 0 4. Xét dấu d2 L tại điểm (x0 ; y0 ) mà (x0 ; y0 ; λ0 ) là nghiệm của hệ phương trình ở bước 3. • Nếu d2 L(x0 ; y0 ; λ0 ) < 0zmax = f (x0 ; y0 ) • Nếu d2 L(x0 ; y0 ; λ0 ) > 0zmin = f (x0 ; y0 ) Để nắm vững phương pháp trên ta quan sát bài toán đơn giản sau: 1 Joseph-Louis Lagrange (1736-1813) là nhà toán học và thiên văn học người Pháp. 1
  2. 1 Cho hai số thực x, y thỏa mãn điều kiện x + y = 10. Tìm giá trị nhỏ nhất của biểu thức: f (x; y) = x2 + y 2 Giải Bước 1: Tìm cực tiểu đối với f (x; y) = x2 + y 2 thỏa mãn điều kiện ϕ(x; y) = x + y − 10 = 0 Bước 2: L(x; y; λ) = x2 + y 2 + λ(x + y − 10) ∂f −λ Bước 3: = 2x + λ = 0 ⇔ x = ∂x 2 ∂f −λ = 2y + λ = 0 ⇔ y = ∂y 2 ∂f −λ = x + y − 10 = 0 ⇒ .2 = 10 ⇔ λ = −10 ∂λ 2 Điểm dừng (5; 5; −10) K Bước 4: L00xx = 2; L00yy = 2; L00xy = 0; d2 L(5; 5; −10) = 2(dx2 + dy 2 ) > 0 fmin = f (5; 5) = 52 + 52 = 25 Qua bài toán 1 chúng ta đã phần nào đó nắm được "tư tưởng" phương pháp này. Để hiểu sâu hơn ta tìm hỏi qua các bài toán khó hơn sau đây. 2 Nếu a và b là các số thực dương thỏa mãn a14 + b14 = 2 . Chứng minh rằng: 5a2 3b3 + 2 ≥8 b a Giải a2 b3 Thiết lập hàm Lagrange L(a, b) = 5 + 3 2 − λ(a14 + b14 − 2). b a Điểm cực trị là nghiệm của hệ: ∂L 10a b3    = − 6 3 − 14a13 λ = 0  ∂a  b a 2 ∂L −5a2 b = 2 + 9 2 − 14b13 λ = 0 T  ∂b b a    a14 + b14 = 2 a Đặt x = , ta quy ước mẫu số bằng 0 thì tử bằng 0 đặt P (x) = 5x18 − 9x14 + 10x4 − 6 = 0. b P (x) = (x2 − 1)Q(x), với Q(x) không có nghiệm thực. Vậy a, b > 0 khi x = 1, điểm cực trị tại a = b = 1,ta có giá trị nhỏ nhất bằng 8. 3 (British Mathematical Olympiad 1986) Cho a, b, c thực thỏa a + b + c = 0 và a2 + b2 + c2 = 6. Tìm giá trị lớn nhất của biểu thức: A = a2 b + b 2 c + c 2 a 2π 4π 8π Bài toán này dấu bằng xảy ra khá đặc biệt khi x = 2cos ; y = 2cos ; z = 2cos . 9 9 9 Chính vì thế nên bài toán gây khó dễ cho các phương pháp ta đã biết, thậm chí là phương pháp mạnh như S.O.S, UMV,.... Trong quyển sử dụng phương pháp Cauchy − Schwarz để chứng minh bất đẳng thức anh Cẩn có một lời giải khá độc đáo như sau: T Giải Xét hàm nhân tử Lagrange như sau: f (a; b; c) = a2 b + b2 c + c2 a + λ1 (a + b + c) + λ2 a2 + b2 + c2 − 6  Các điểm cực trị là nghiệm hệ phương trình: ∂f ∂f ∂f   ∂a = ∂b = ∂c = 0    a+b+c=0   2 a + b2 + c 2 = 6 2
  3.    2ab + c2 + λ1 + 2λ2 a = 0  2  2bc + a + λ1 + 2λ2 b = 0    ⇔ 2ca + b2 + λ1 + 2λ2 c = 0  a+b+c=0       a2 + b 2 + c 2 = 6 Cộng vế với vế của phương trình thứ nhất, thứ hai, thứ ba ta được (a + b + c)2 + 3λ1 + 2λ2 (a + b + c) = 0 ⇔ λ1 = 0 Đến đây ta được   2ab + c2 + 2λ2 a = 0 K   2  2bc + a + 2λ2 b = 0    2ca + b2 + 2λ2 c = 0  a+b+c=0       a2 + b 2 + c 2 = 6 Từ đây ta có 2ab + c2 2bc + a2 2ca + b2 = = = −2λ2 a b c Đây là điều kiện của dấu đẳng thức trong bất đẳng thức Cauchy − Schwarz nên ta sẽ chứng minh bài toán như sau: Theo bất đẳng thức Cauchy − Schwarz ta có [a(2ab + c2 ) + b(2bc + a2 ) + c(2ac + b2 )]2 ≤ (a2 + b2 + c2 )[(2ab + c2 )2 + (2bc + a2 )2 + (2ac + b2 )2 ] Mặt khác ta có: a(2ab + c2 ) + b(2bc + a2 ) + c(2ac + b2 ) = 3(a2 b + b2 c + c2 a) và T 2 2 (2ab + c2 ) = 2(ab + bc + ca)2 + (a2 + b2 + c2 ) = 54 P Nên do đó A = a2 b + b2 c + c2 a ≤ 6. 2π 4π 8π Dấu "=" xảy ra khi x = 2cos ; y = 2cos ; z = 2cos 9 9 9 4 Cho x, y, z thực thỏa x + y + z = 0 và x2 + y 2 + z 2 = 2. Tìm GTLN, GTNN của biểu thức: P = x3 + y 3 + z 3 Giải Đặt f (x; y; z) = x3 + y 3 + z 3 + λ1 (x + y + z) + λ2 (x2 + y 2 + z 2 − 2) Điểm cực trị là nghiệm của hệ ∂f ∂f ∂f   ∂x = ∂y = ∂z = 0    x+y+z =0   2 x + y2 + z2 = 2 Hay  T   3x2 + λ1 + 2λ2 x = 0  2  3y + λ1 + 2λ2 y = 0    3z 2 + λ1 + 2λ2 z = 0  x+y+z =0       x2 + y 2 + z 2 = 2 Cộng lại ta có 3(x2 + y 2 + z 2 ) + 3λ1 + 2λ(x + y + z) = 0 ⇔ λ1 = −2 Thay vào hệ trên ta có  2  3x + 2λ2 x − 2 = 0  3y 2 + 2λ2 y − 2 = 0   2 3z + 2λ2 z − 2 = 0 3
  4. Ở đây ta quy ước nếu mẫu bằng 0 thì tử bằng 0 3x2 − 2 3y 2 − 2 3z 2 − 2 = = = −2λ2 x y z Áp dụng bất đẳng thức Cauchy-Schwarz ta có: [x(3x2 − 2) + y(3y 2 − 2) + z(3z 2 − z)]2 ≤ (x2 + y 2 + z 2 )[(3x2 − 2)2 + (3y 2 − 2)2 + (3z 2 − 2)2 ] = 2.[9(x4 + y 4 + z 4 ) − 12] = 12 Do x4 +y 4 +z 4 = (x2 +y 2 +z 2 )2 −2[(xy)2 +(yz)2 +(xz)2 ] = 4−2[(xy +xz +yz)2 −2xzy(x+y√ +z)] √ Mà K 2 2 2 2 (x + y + z) − (x + y + z ) 2 3 2 3 xy + xz + yz = = −1 Do đó. x4 + y 4 + z 4 = 2 Vậy − ≤P ≤ √ 2   3 3 2 3 1 1 2 • min P = − khi (x; y; z) = √ ; √ ; − √ và các hoán vị. √3  3 3 3 2 3 1 1 2 • max P = khi (x; y; z) = − √ ; − √ ; √ và các hoán vị. 3 3 3 3 5 Cho a; b; c thực thỏa mãn a2 + b2 + c2 + abc = 4. Tìm giá trị nhỏ nhất của biểu thức: D =a+b+c Giải Đặt f (a; b; c) = a + b + c; g(a; b; c) = a2 + b2 + c2 − 4 + abc L = f − λg = a + b + c − λ(a2 + b2 + c2 + abc − 4) Điểm cực trị là nghiệm của hệ T ∂L 1     = 1 − λa − λbc = 0   λ=  ∂a     2a + bc ∂L 1  = 1 − λb − λac = 0 ⇒ λ=  ∂b  2b + ac   ∂L   1 = 1 − λc − λba = 0  λ=    ∂c 2c + ba 1 1 Do = ⇔ (a − b)(2 − c) = 0 . Tương tự ta có (b − c)(2 − a) = 0 và (c − a)(2 − b) = 0 2a + bc 2b + ac Nếu a = b = 2 từ a2 + b2 + c2 + abc = 4 ta tìm được c = −2 và vì vậy a + b + c = 6 Nếu a = b = c 6= 2 ta có 3a2 + a3 = 4 ⇔ (a − 1)(a + 2)2 = 0 Vậy ta có a = b = c = 1 hoặc a = b = c − 2 và từ a + b + c = 3 hoặc a + b + c = −6 ta có giá trị nhỏ nhất của D = −6 6 Cho a, b, c thực dương thỏa a + b + c + d = 1. Chứng minh rằng: 1 176 abc + bcd + cda + dab ≤ + abcd T 27 27 Giải 1 Đặt f = abc + bcd + cda + dab ta sẽ chứng minh f ≤ . 27 Đặt g = a + b + c + d − 1 Ta thiết lập hàm Lagrange 126 L = f − λg = abc + bcd + cda + dab − abcd − λ(a + b + c + d − 1) 27 4
  5. Ta có:  ∂L 176 = bc + cd + db − bcd − λ = 0   ∂a 27     ∂L = ac + cd + da − 176 acd − λ = 0    ∂b 27 ∂L 176   = ba + ad + db − bad − λ = 0 ∂c 27   ∂L 176      = bc + ca + ab − bca − λ = 0 ∂d 27 Từ hệ ta tìm được: 176 176 176 176 λ = bc + cd + db − bcd = ac + cd + da − acd = ab + bd + da − abd = bc + ac + ab − abc. 27 27 27 27 K 176 176 127 Từ bc + cd + bd − bcd = ac + cd + da − acd ta có (b − a)(c + d − cd) = 0. Thiết lập tương 27 27 27 tự: 127 (b − c)(a + d − ad) = 0 27 176 (b − d)(a + c − ac) = 0 27 176 (a − c)(b + d − bd) = 0 27 1 Giải phương trình này ta có a = b = c = d và từ a + b + c + d = 1 nên a = b = c = d = . Vậy   4 1 1 1 1 1 f ; ; ; = . Từ đây ta có điều cần chứng minh.  4 4 4 4 27 7 Cho a, b, c thực thỏa a + b + c > 0. Chứng minh rằng: a3 + b3 + c3 ≤ (a2 + b2 + c2 )3/2 + 3abc T Trong bài toán này ta sẽ tìm cách đặt ẩn thích hợp để tạo thêm điều kiện ràng buộc cho các biến để sử dụng phương pháp nhân tử Lagrange. Giải Đặt a b c x= √ ; y=√ ; z=√ 2 2 a +b +c 2 2 a +b +c2 2 a + b2 + c 2 2 Bất đẳng thức cần chưng minh viết lại thành x3 + y 3 + z 3 ≤ (x2 + y 2 + z 2 )3/2 + 3xyz với x2 + y 2 + z 2 = 1 Đặt f = a3 + b3 + c3 − 3abc; g = a2 + b2 + c2 − 1 Ta thiết lập hàm Lagrange T L = f − λg = a3 + b3 + c3 − 3abc − λ(a2 + b2 + c2 − 1) Ta có hệ sau: ∂L    = 3a2 − 3bc − 2λa = 0  ∂a   ∂L = 3b2 − 3ac − 2λb = 0  ∂b  ∂L = 3c2 − 3ba − 2λc = 0    ∂a 2 3(a − bc) 3(b2 − ac) 3(c2 − ab) λ= = = 2a 2b 2c 5
  6. Từ trên ta có 3(a2 − bc) 3(b2 − ac) = ⇔ (a − b)(ab + bc + ac) = 0 2a 2b Tương tự ta có (b − c)(ab + bc + ac) = 0 và (c − a)(ab + bc + ac) = 0. Giải phương trình này ta thu được a = b = c hoặc ab + bc + ac = 0. Nếu a = b = c thì f (a; a; a) = 0 < 1 Nếu ab + bc + ac = 0 thì (a + b + c)2 = a2 + b2 + c2 + 2(ab + bc + ac) = 1 Và từ a + b + c > 0 ta có a + b + c = 1 Vì vậy f (a; b; c) = a3 + b3 + c3 − 3abc = (a + b + c)(a2 + b2 + c2 − ab − bc − ac) = 1 K Phép chứng minh hoàn tất.  8 (China TST -2004)Cho a, b, c, d thực dương thỏa abcd = 1. Chứng minh rằng: 1 1 1 1 2 + 2 + 2 + ≥1 (1 + a) (1 + b) (1 + c) (1 + d)2 Giải 1 1 1 1 Đặt f (a; b; c; d) = 2 + 2 + 2 + và g(a; b; c; d) = abcd − 1 (1 + a) (1 + b) (1 + c) (1 + d)2 Ta thiết lập hàm Lagrange 1 1 1 1 L = f − λg = 2 + 2 + 2 + − λ(abcd − 1). (1 + a) (1 + b) (1 + c) (1 + d)2 Ta có hệ sau ∂L −4 λ −4a   = 2 − =0 λ=     ∂a (1 + a) a (1 + a)2       −4 −4b T     ∂L λ   = 2 − =0  λ=     ∂b (1 + b) b (1 + b)2  ∂L −4 λ ⇒ −4c 2 −   = =0   λ= ∂c c (1 + c)2     (1 + c)   −4d     ∂L −4 λ   = 2 − =0  λ=     ∂c c (1 + d)2  (1 + c) Từ hệ ta thu được −4a −4b −4c −4d 2 = 2 = 2 = =λ (1 + a) (1 + b) (1 + c) (1 + d)2 Ta có: (a − b)(1 − ab) = 0; (a − c)(1 − ac) = 0; (a − d)(1 − ad) = 0 (b − c)(1 − bc) = 0; (b − d)(1 − bd) = 0; (c − d)(1 − cd) = 0 Giải phương trình ta thu được a = b = c = d và từ abcd = 1 nên a = b = c = d = 1 vì vậy ta có T 1 1 1 1 f (1; 1; 1; 1) = + + + =1 4 4 4 4 Từ 1 1 1 1 4 1 5 f (1; 1; ; 2) = + + + = + > 1 2 4 4 9 9 2 9 Nên theo phương pháp nhân tử Lagrange ta kết luận được f (a; b; c; d) ≥ 1. Vậy bài toán được chứng minh.  6
  7. 9 Cho a, b, c, d là những số  a + b + c + d = 4. Chứng minh rằng:  thực dương thỏa 1 1 1 1 27 + + + ≥ 9 (a3 + b3 + c3 + d3 ) + 8. a b c d Giải Ta thiết lập hàm Lagrange lấy vế trái trừ vế phải, và đặt g là a + b + c + d − 4 = 0. Ta dễ dàng thấy ∂f ∂( ) ∂a < 0,nên nó tồn tại giá trị cực tiểu. ∂a ∂f ∂g Nên nó tồn tại hằng số λ thỏa mãn =λ× . ∂a ∂a 1 ⇔ −27( 2 + a2 ) = λ a K 1 Từ trên ta có a, b, c, d = k hoặc . k 1) Nếu a, b, c, d = k thì k = 1, ta tìm được f > 0. 1 1 2) Nếu a, b, c = k và d = , thì k = và f = 0. k 3 1 3) Nếu a, b = k và c, d = , thì k = 1, và f > 0. k Từ 1) 2) 3) ta có điều cần chứng minh.  10 Cho a, b, c, d thực dương thỏa a + b + c + d + abcd = 5. Chứng minh rằng: 1 1 1 1 + + + ≥4 a b c d Giải 1 1 1 1 Đặt f (a; b; c; d) = + + + và g(a; b; c; d) = a + b + c + d + abcd − 5 = 0 a b c d Ta thiết lập hàm Lagrange 1 1 1 1 L = f − λg = + + + − λ(a + b + c + d + abcd − 5) T a b c d  ∂L 1 =− − λ(1 + bcd) = 0   ∂aa2      ∂L1 = − 2 − λ(1 + acd) = 0   ∂bb ∂L1   = − 2 − λ(1 + bad) = 0 ∂cc   ∂L1      = − 2 − λ(1 + bca) = 0 ∂dd −1 −1 −1 −1 λ= 2 = 2 = 2 = a (1 + bcd) b (1 + acd) c (1 + abd) 1 + d2 (acb) Từ 2 phương trình đầu ta có: a2 (1 + bcd) = b2 (1 + cad) ⇔ (a − b)(a + b + abcd) = 0 T Từ a + b + abcd > 0 ta có a = b. Tương tự ta có a = c = d nên a = b = c = d Sử dụng giả thiết a+b+c+d+abcd = 5 ta thu được a4 +4a−5 = 0 ⇔ (a−1)(a3 +a2 +a+5) = 0 giải phương trình này ta tìm được a = 1. Do đó a = b = c = d = 1. Nên f (1, 1, 1, 1) = 1 + 1 + 1 + 1 = 4 vậy ta có điều cần chứng minh.  11 Cho a, b, c thực dương thỏa a + b + c = 1. Chứng minh rằng: 7(ab + bc + ac) ≤ 9abc + 2 Giải Đặt f (a; b; c) = 7(ab + cb + ac) − 9abc − 2 , g(a; b; c) = a + b + c − 1 Ta thiết lập hàm Lagrange L = f − λg = 7(ab + bc + ac) − 9abc − 2 − λ(a + b + c − 1) 7
  8. Điểm cực trị là nghiệm của hệ ∂L   = 7(b + c) − 9bc − λ = 0   λ = 7(b + c) − 9bc    ∂a  ∂L  = 7(c + a) − 9ac − λ ⇒ λ = 7(a + c) − 9ac  ∂b   ∂L = 7(a + b) − 9ab − λ = 0 λ = 7(b + a) − 9ba     ∂c Nên 7(b + c) − 9bc = 7(c + a) − 9ac = λ ⇔ (b − a)(7 − 9c) = 0(1) Thiết lập tương tự ta cũng có (c − a)(7 − 9a) = 0(2) và (a − c)(7 − 9b) = 0(3) giải phương trình ta K 1 có a = b = c = và 3 21 9 f (a; b; c) = 7(ab + bc + ac) − 9abc − 2 = − −2=0 9 27 7 14 Từ (1) nếu a = b và b 6= c thì từ phương trình trên ta có a = = b và a + b = > 1 điều này mâu 9 9 thuẫn do a + b < a + b + c = 1 Nếu 7 − 9c = 0 thì ta không thể có 7 − 9a = 0 hoặc 7 − 9b = 0 nên từ (2); (3) ta phải có b = c và 7 a = c nên a = b = c = . 9 Vậy giá trị nhỏ nhất L = 0 hay 7(ab + ac + bc) ≤ 9abc + 2 12 Cho a, b, c thực thỏa a2 + b2 + c2 + d2 = 1 chứng minh rằng: A = a3 + b3 + c3 + abc + bcd + cda + dab ≤ 1 Giải T Ta thiết lập hàm Lagrange L = a3 + b3 + c3 + d3 + abc + bcd + cda + dab − 1 − λ(a2 + b2 + c2 + a2 − 1) Điểm cực trị là nghiệm của hệ  ∂f   = 3a2 + bc + cd + bd − 2λa = 0 ∂a   ∂f   = 3b2 + ac + cd + ad − 2λb = 0      ∂b ∂f = 3c2 + ba + bd + ad − 2λc = 0   ∂c ∂f   = 3d2 + bc + ca + ba − 2λd = 0       ∂d a2 + b 2 + c 2 + d 2 = 1  T 3a2 + bc + ac + bd    2λ = a   2     2λ =  3b + ac + cd + ad ⇒ b 3c2 + ba + bd + ad 2λ =      2 c 3d + ca + bc + ab    2λ =  d Ta quy ước nếu mẫu bằng 0 thì tử bằng 0 Từ đây ta có: 3a2 + bc + ac + bd 3b2 + ac + cd + ad = a b 8
  9. 3c2 + ba + bd + ad 3d2 + ca + bc + ab = = = 2λ c d Theo bất đẳng thức Cauchy − Schwarz ta có: X X 9V T 2 = [ a(3a2 + bc + cd + db)]2 ≤ (a2 + b2 + c2 )[ (3a2 + bc + cd + db)2 ] = 9(a2 + b2 + c2 + d2 )2 − 2(a2 b2 + b2 c2 + c2 a2 + d2 a2 + d2 b2 + c2 d2 ) − abc(a + b + c) −bcd(b + c + d) − cda(c + d + a) − bab(b + d + a) ≤ 9 Thật vậy bất đẳng thức cuối tương đương: K 2 a2 b2 + b2 c2 + c2 a2 + d2 a2 + d2 b2 + c2 d2 ≥ abc (a + b + c)+bcd (b + c + d)+cda (c + d + a)+dab (d + b + a)  Luôn đúng do AM − GM . (bộ [2; 2; 0; 0]  [2; 1; 1; 0] Bất đẳng thức được chứng minh.  12 (IMO Shortlist 2007) Cho a1 ; a2 ; ...a100 ≥ 0 thỏa a21 + a22 + ... + a2100 = 1 chứng minh rằng: a21 a2 + a22 a3 + ... + a2100 a1 Giải Để thuận tiện biến đổi ta đặt a101 = a1 ; a102 = a2 và S = a21 a2 + a22 a3 + ... + a2100 a1 Áp dụng bất đẳng thức Cauchy-Schwarz: " 100 #2 100 !  100 2  X X X 9S 2 = ak+1 (ak 2 + 2ak+1 ak+2 ) ≤ ak+1 2  (ak 2 + 2ak+1 ak+2 )  k=1 k=1 k=1 100 100 2 2 X X 2 = (ak + 2ak+1 ak+2 ) = (ak 4 + 4ak 2 ak+1 ak+2 + 4ak+1 2 ak+2 2 ) T k=1 k=1 Mặt khác theo bất đẳng thức AM-GM ta có 4ak 2 ak+1 ak+2 ≤ 2a2k (a2k+1 + a2k+1 ) Từ trên ta có 100 X 100 X 2 a4k 2a2k (a2k+1 a2k+2 ) 4a2k+1 a2k+2  4 ak + 6a2k+1 a2k + 2a2k a2k+2    9S ≤ + + + = k=1 k=1 Sử dụng các đánh giá đơn giản 100 100 !2 X  4 X ak + 2a2k+1 a2k + 2a2k a2k+2 ≤ a2k  =1 k=1 k=1 T 100 50 ! 50 ! X X X a2k a2k+1 ≤ a22i−1 a22j k=1 i=1 j=1 Ta được: ! ! !2 50 X 50 X 50 X 50 X 9S 2 ≤ 1 + 4 a22i−1 a22j ≤1+ a22i−1 + a22j =2 i=1 j=1 i=1 j=1 √ 2 12 Từ đây suy ra S ≤ 3 < 25 . Vậy bài toán được chứng minh  9
  10. BÀI TẬP Bài 1: Cho x, y, z thực thuộc đoạn [0; 1], chứng minh rằng x2 + y 2 + z 2 ≤ 1 + x2 y + y 2 z + z 2 x Bài 2: Cho x; y; z thực dương. Chứng minh rằng: x y z + + ≤1 1 + x + xy 1 + y + yz 1 + z + xz K Bài 3: (USAMO -1997)Cho a; b; c; d thực dương. Chứng minh rằng:   r r 2 1 1 1 1 1 p q 25 ≤ (a + b + c + d + e) + + + + ≤ 25 + 6 − a b c d e q p Bài 4: (USAMO 2001) Cho a, b, c thực dương thỏa a2 + b2 + c2 + abc = 4 chứng minh rằng ab + bc + ac − abc ≤ 2 Bài 5: (1999 Canada Math Olympiad) Cho x, y, z là những số thực không âm thỏa x + y + z = 1. Chứng minh rằng 4 x2 y + y 2 z + z 2 x ≤ 27 Bài 6: (Bất đẳng thức Schur) Cho a, b, c ≥ 0.Chứng minh a3 + b3 + c3 + 3abc ≥ ab(a + b) + bc(b + c) + ca(c + a) T Bài 7: (Romanian IMO-BMO TST -2007)Cho các số thực a1 ; a2 ; ...an ; b1 ; b2 ; ...bn (n ≥ 2) thỏa mãn a21 + a22 + ... + a2n = b21 + b22 + ... + b2n = 1; a1 b1 + a2 b2 + ... + an bn = 0. Chứng minh rằng (a1 + a2 + ... + an )2 + (b1 + b2 + ... + bn )2 ≤ n Bài 8: Cho a1 , a2 , ...., an > 0 và a1 .a2 ....an = 1.Chứng minh 1 1 1 n no + + ...... + ≥ min 1; k ∀k ≥ 0 (1 + a1 )k (1 + a2 )k (1 + an )k 2 Bài 9: (VMEO II)Cho a, b, c, x, y, z thỏa ax + by + cz = xyz. Chứng minh q p x + y + z ≥ 4(a + b + c) + 8(ab + bc + ca) Bài 10: Cho a, b, c > 0 thỏa a2 + b2 + c2 = 3. Chứng minh: T 2(a2 b + b2 c + c2 a) + 15 ≥ 3(a + b + c) + 4(ab + bc + ca) Bài 11: Cho a, b, c ∈ R thỏa 2a3 + 2b3 + c3 = 4.Tìm GTLN và GTNN của c2 − 2ab Bài 12:Cho a, b, c ≥ 0, a + b + c = 3. Chứng minh rằng: a4 + b4 + c4 + ab + bc + ca ≥ 6 10
  11. Bài viết xin kết thúc tại đây. Nhân đây, tôi xin cám ơn anh Hoàng Quốc Việt-sinh viên trường Đại học Bách Khoa Hà Nội đã có những ý kiến đóng góp để hoàn thiện chuyên đề này. Việc biên soạn không thể tránh khỏi những thiếu sót, rất mong những ý kiến đóng góp của các bạn để chuyên đề được hoàn thiện hơn! Mọi đóng góp xin gửi về địa chỉ: Tài liệu [1] Võ Quốc Bá Cẩn -Trần Quốc Anh Sử dụng phương Pháp Cauchy-Schwarz để chứng minh bất đẳng thức. K [2] www.diendantoanhoc.net [3] www.artofproblemsolving.com/Forum [4] Ngô Thành Phong Giáo trình giản yếu giải tích toán học - ĐHKHTN -ĐHQG TPHCM [5] Zdravko Cvetkovski Inequalities theorems, techniques and Selected Problems T T 11


Page 2

YOMEDIA

Tài liệu "Phương pháp nhân tử lagrange: Method of lagrange multipliers" trình bày về các bước cơ bản của phương pháp nhân tử lagrange, định nghĩa, những câu hỏi bài tập về phương pháp nhân tử lagrange,...

19-12-2015 236 14

Download

Bài tập về phương pháp Lagrange

Giấy phép Mạng Xã Hội số: 670/GP-BTTTT cấp ngày 30/11/2015 Copyright © 2009-2019 TaiLieu.VN. All rights reserved.